Weaken

This topic has expert replies
Moderator
Posts: 772
Joined: Wed Aug 30, 2017 6:29 pm
Followed by:6 members

Weaken

by BTGmoderatorRO » Sun Oct 29, 2017 10:41 am
Political Analyst: Because our city is a border city, illegal immigration is an important issue in the current race for mayor. Of the two candidates for mayor, one supports a plan that would attempt to deport the city's 9,000 illegal immigrants and the other does not. Surveys consistently show that about 60% of the city's residents are opposed to the plan, while about 35% are in support of the plan. Therefore, the candidate who does not support the plan will win the election for mayor.
All of the following statements weaken the analyst's argument, EXCEPT:

(A) In the city at issue, most voters make their voting decisions based on the candidates' positions on abortion.
(B) Of the 35% of residents who support the plan, some are willing to consider alternate plans for addressing illegal immigration.
(C) Many of the residents who oppose the plan are not registered voters.
(D) The candidate who supports the plan is the incumbent mayor, and has been elected to four consecutive terms despite taking controversial positions on many important issues.
(E) Just under 30% of the city's residents are illegal immigrants who cannot vote.
OA is b

Can any expert explain what the correct option is? Thanks

User avatar
GMAT Instructor
Posts: 555
Joined: Wed Oct 04, 2017 4:18 pm
Thanked: 180 times
Followed by:12 members

by EconomistGMATTutor » Fri Nov 03, 2017 9:03 am
The conclusion: The candidate who does not support the plan will win the election for mayor.

The evidence: 60% of the city's residents are opposed to the plan, while about 35% are in support of the plan.

The assumption: The survey accurately reflects how the election will turn out. There are many ways that this can be attacked.

Choice A: If voters don't care as much about the immigration issue, then the survey does not necessarily reflect how they'll vote. This weakens the conclusion.
Choice B: This is the correct choice. Some of the people who support the plan are open to other plans. And so? This has no effect on the conclusion. Since this is an EXCEPT question, it's correct.
Choice C: If they opposers don't vote, the survey results do not reflect how the election will turn out. This weakens the conclusion.
Choice D: This implies that the mayor, despite being a supporter of the plan, is likely to win. This weakens the conclusion.
Choice E: The survey was of residents. If a substantial percentage of them are illegal immigrants, who are likely to oppose the plan, the survey does not reflect how the vote will turn out. This weakens the conclusion.

I'm available if you'd like any follow up.
GMAT Prep From The Economist
We offer 70+ point score improvement money back guarantee.
Our average student improves 98 points.

Image